What is the probability of rolling a two or not rolling a 2 using a regular six sided number cube

Answers

Answer 1

Answer:

It's a 1/6 chance.

Step-by-step explanation:


Related Questions

how many pairs of prime numbers have a sum of 24?​

Answers

Answer:

Three pairs of prime numbers exist: 5 + 19 = 24, 7 + 17 = 24, 11 + 13 = 24.

Step-by-step explanation:

hope this helps :)

Answer:

Three: 5 + 19 = 24, 7 + 17 = 24, 11 + 13 = 24.

Step-by-step explanation:

A 2 pound package of shredded cheese is $6.98 A 1/2 pound package of shredded cheese is $1.98 what is the difference in the cost per pound between the larger and smaller packages of shredded cheese

Answers

Answer:

$3.89

Step-by-step explanation:

what fraction of 60 is 20

Answers

Step-by-step explanation:

60%20 is 3 is the answer.

The value of fraction will be 1/3.

What is mean by Fraction?

A fraction is a part of whole number, and a way to split up a number into equal parts. Or, A number which is expressed as a quotient is called fraction. It can be written as the form of p : q, which is equivalent to p / q.

Given that;

The expression is,

''A fraction of 60 is 20.''

Now,

Let the value of fraction = x

So, We can formulate;

⇒ x of 60 = 20

⇒ x × 60 = 20

⇒ x = 20/60

⇒ x = 2/6

⇒ x = 1/3

Thus, The value of fraction = 1/3

Learn more about the fraction visit:

https://brainly.com/question/5454147

#SPJ2

An Ice Cream Factory makes 3:1 tubs of ice cream to boxes of ice cream bars. If the factory makes 165 tubs of ice cream in a day, how many boxes of ice cream bars were made?​

Answers

Answer:

55 boxes of ice cream

Step-by-step explanation:

Tubs of ice cream : boxes of ice cream = 3 : 1

If the factory makes 165 tubs of ice cream in a day, how many boxes of ice cream bars were made?

Boxes of ice cream made in a day = x

Tubs of ice cream : boxes of ice cream = 165 : x

Equate both ratios to find x

3 : 1 = 165 : x

3/1 = 165/x

Cross product

3 * x = 1 * 165

3x = 165

x = 165/3

x = 55

Boxes of ice cream made in a day = 55

Help I’ll give Brainliest
What is the solution to the system of equations graphed below

Answers

It’s answer b, I hope this helped :)

Answer:

D)  (0, -4)

Step-by-step explanation:

the solution is the point at which the two lines intersect

PLS I NEED HELP WITH MY EXAM
a sequence is represented below.
{–5, –25, –125, –625, -3125, ...}
Which recursive notation could be used to represent the sequence?
F. a1 = –5, an = 5an-1
G. a1 = 5, an = –5an–1
H. a1 = –5, an = an-1+ 5
J. a1 = 5, an = an–1 – 5

Answers

Answer:

F. a1 = –5, an = 5an-1

Step-by-step explanation:

a1 = -5                    <---- Answer

r = -25/-5 = 5

an =  r(an - 1)

an =  5(an - 1) <---- Answer

Hope this helps!

g

Step-by-step explanation:

A. -4

B. 4

C. -1/4

D. 1/4

Answers

Answer:

D

Step-by-step explanation.

The slope is the change in y over the change in x. If you look at the change in y, it goes from 2 to 3, and 3-2=1. If you look at the change in x, it goes from -1 to 3, and 3-(-1)=4 (if you subtract a negative it adds the number instead). IF you put the change in y over the change in x, you get 1/4, which is D

Gwen is going to Turkey. The exchange rate is £1 = 5.8427 lira. She changes £424 to lira. How
many lira will she get to the nearest lira?

Answers

______________________________

= £424 × 5.8427lira= 2,477.31 liraGwen Will Get 2,477.31 lira

______________________________

answers for the 2 boxes please:)​

Answers

Answer:

(1,2)

Step-by-step explanation:

Eliminate y-term

[tex](x + 2x) + (4y - 4y) = 9 - 6 \\ 3x = 3 \\ x = \frac{3}{3} \\ x = 1[/tex]

Substitute x = 1 in any given equations.

I choose to substitute in the first equation.

[tex]x + 4y = 9 \\ 1 + 4y = 9 \\ 4y = 9 - 1 \\ 4y = 8 \\ y = \frac{8}{4} \\ y = 2[/tex]

Answer Check

Substitute both x-value and y-value in any given equations.

[tex]x + 4y = 9 \\ 1 + 4(2) = 9 \\ 1 + 8 = 9 \\ 9 = 9[/tex]

The first equation is true. However, we have to check the second equation as well. (Although the first true equarion has already raised the high possibility of right answer.)

[tex]2x - 4y = - 6 \\ 2(1) - 4(2) = - 6 \\ 2 - 8 = - 6 \\ - 6 = - 6[/tex]

Both equations are true for (1,2). Thus the answer is (1,2)

Konnor earns $10 an hour mowing lawns. He spends $17 a day on fuel. Konnor's earnings can be represented by the linear relation $10h - $17 = E, where h represents hours worked and E is Konnor's total earnings. If he works 9 hours a day, what are his earnings?

Answers

10 an hour it is if you doing

A rectangle has a length of (5r+8) inches and a width of (2r + 4) inches. What is the area of the rectangle?

Option 1: 10r^2 - 36r + 32
Option 2: 10r^2 + 36r + 32
Option 3: 10r^2 + 32
Option 4: 10r^2 - 32

Answers

Answer:

Option 1: 10r^2 - 36r + 32

Step-by-step explanation:

Option 1: 10r^2 - 36r + 32

y + 7x=50
14x - 5y = -28
one solution
no solution
Infinite solutions

Answers

Answer:

Your answer is in the screenshot! :)

Step-by-step explanation:

1. Solve for yy in y+7x=50y+7x=50. y=50-7x

y=50−7x

2. Substitute y=50-7xy=50−7x into 14x-5y=-2814x−5y=−28. 49x-250=-28

49x−250=−28

3. Solve for xx in 49x-250=-2849x−250=−28. x=\frac{222}{49}

x= 49 222 ​

4. Substitute x=\frac{222}{49}x= 49 222 ​ into y=50-7xy=50−7x.

y=\frac{128}{7} y= 7 128 ​

5. Therefore, \begin{aligned}&x=\frac{222}{49}\\&y=\frac{128}{7}\end{aligned} ​ x= 49 222 ​ y= 7 128 ​ ​Hope this helped!! :))

Find the value of x.

Answers

Answer:

x=14

Step-by-step explanation:

17=x+3

x=14

Answer:

c, x=14

Step-by-step explanation:

The sides of a rhombus are parallel to the opposite one.

Therefore, x + 3 = 17

x=14

Which choices are real numbers. Check all that apply. A. (-1024)^1/4. B. (-131072)^1/16. C. (-256)^1/9. D. (-531441)^1/13

Answers

Answer:

B and C

Step-by-step explanation:

Because it is raised to the ^Power of 13. This choice makes the most sense.

The real numbers are [tex](-256)^{1/9}[/tex] and [tex](-531441)^{1/13}[/tex]. Then the correct options are C and D.

What is a real number?

All the numbers are real number.

If the number is having iota (i), then the number becomes complex number.

A. [tex](-1024)^{1/4} = 4 * (-2^2)^{1/4} = 4\sqrt2 \iota[/tex]

B. [tex](-131072)^{1/16} = 2*(-2)^{1/16} = 2 \sqrt[16]{2} \iota[/tex]

C. [tex](-256)^{1/9} = -1.85[/tex]

D. [tex](-531441)^{1/13} = -2.76[/tex]

Then the real numbers are [tex](-256)^{1/9}[/tex] and [tex](-531441)^{1/13}[/tex].

Then the correct options are C and D.

More about the real number link is given below.

https://brainly.com/question/551408

#SPJ5

Asap? ABCD is a parallelogram. which statement is true

Answers

Step-by-step explanation:

The Value of Y is three times than a multiple of x.

no one is helping me please help me due today. determine whether the quadrilateral is a parallelogram by using the indicated method​

Answers

Answer:  ( − 5 , − 6 ) , ( 5 , 2 ) , ( 4 , − 4 ) , ( − 6 , − 12 )

Maths question.... I’m really bad at maths

Answers

Step-by-step explanation:

Cylinder's Total Surface area = 3πr²

Here r=10 cm

3π10²

300π

Hope this helps you.

Answer:

200π

Step-by-step explanation:

4(10)²π ÷ 2 since it is only half a sphere

400π ÷ 2 = 200π

1) Jayden states that based on the Congruent Supplements Theorem, if
mZ1 +mZ2 = 90° and if mZ1 +mZ3 = 90°, then 22 =23. What is
the error in Jayden's reasoning?

Answers

Answer:

The congruent supplements theorem states that 2 angles are supplementary if they add up to 180 degrees, not 90 degrees.

He should have stated the complimentary theorem instead.

Step-by-step explanation:

The correct statement will be,'if ∠1 +∠2 = 180° and if ∠1 +∠3 = 180°, then ∠2 =∠3'.

Given:

∠1 +∠2 = 90° and if ∠1 +∠3 = 90°, then ∠2 =∠3

To find:

The error in Jayden's reasoning

Solution:

According to Congruent Supplements Theorem, the sum of supplementary angles is equal to 180°, not 90°.

[tex]\angle 1 +\angle 2 = 180^o\\\angle 1 +\angle 3 = 180^o\\\text{From this, we can conclude that}:\\\angle 2 = \angle 3[/tex]

The error in Jayden's reasoning was 90°.

The correct statement will be,'if ∠1 +∠2 = 180° and if ∠1 +∠3 = 180°, then ∠2 =∠3'.

Learn more about Congruent Supplementary Theorem here:

brainly.com/question/2506908?referrer=searchResults

brainly.com/question/18283981?referrer=searchResults

Choose the correct comparison for the numbers below.

0.85, 3/4, 91%

a. 3/4 > 0.85 > 91%

c. 91% > 3/4 > 0.85

d. 0.85 < 3/4 < 91%

Answers

Answer:

A

Step-by-step explanation:

In order to compare all three numbers, you must change them all to have the same form, either all decimal form, all fraction, or all percentages.

The easiest ways to compare are decimal and percentage, as they're practically the same thing. For simplicities sake, though, let's turn all the numbers into decimal.

Since 0.85 is already in decimal form, leave it as it is.

3/4 can be turned into a decimal by putting it into a calculator, but intuitively you should know that it's 0.75. If you don't know a fraction intuitively, you just need to make the fraction into a multiple of denominator 100 and take the numerator. For example:

[tex]\frac{3}{4} * \frac{25}{25} = \frac{75}{100}[/tex]

Numerator is 75.0, now simply move the decimal place up to to get the decimal form 0.75.

91% can be changed into a decimal by moving the decimal place left 2 places. 91.0% turns into 0.91.

Now that you can see the numbers, just rearrange them from least to greatest

0.75<0.85<0.91

equal to A

Clare is painting some doors that are all the same size. She used 2 liters of paint to cover 1LaTeX: \frac{3}{5}3 5 doors. How many liters of paint are needed for 1 door?

Answers

Answer:

1 1/4 liters of paint

Step-by-step explanation:

Clare is painting some doors that are all the same size. She used 2 liters of paint to cover 1LaTeX: \frac{3}{5}3 5 doors. How many liters of paint are needed for 1 door?

From the above question, we can deduce that:

2 liters of paint covers 1 3/5 doors

Hence:

1 3/5 doors = 2 liters of paint

1 door = x liters of paint

Cross Multiply

1 3/5 doors × x liters = 2 liters × 1 door

x liters = 2 liters × 1 door/ 1 3/5 doors

x liters = 2 ÷ 1 3/5

x liters = 2 ÷ 8/5

x liters = 2 × 5/8

x liters = 5/4 liters

x liters = 1 1/4 liters of paint

Hence: 1 1/4 liters of paint is needed for 1 door

2. It takes exactly 26 paper clips laid end to end to make a length of 17 7/8 inches.
Calculate the length of each paper clip. Show your reasoning.

Answers

Answer:

Andresiaは13.1マイルの長さのレースを走っています。彼はすでに1.1マイル走っています。アンドレスが走るために残っているマイル数を再確認する

Answer:

WHAT HE SAID Andresiaは13.1マイルの長さのレースを走っています。彼はすでに1.1マイル走っています。アンドレスが走るために残っているマイル数を再確認する

its not even right :D

Step-by-step explanation:

I’ll give the Brainliest to who answers this question with a reasonable explanation.

The population of a small town increased 23% from its population in 2000 to its population in 2003. If the population of the town in 2000 was 25,314, what was its population in 2003?

Answers

Answer:

31136

Step-by-step explanation:

To find the how much an number increased by a percent. Divide the percent by 100. 23/100=0.23.

Then multiply 0.23 by the original number which is 25,314. Which gives us about 5822.

Then since it INCREASED, we are going to add that number it increased to the original. 25314+5822= 31136.

Answer:

Good Luck, hope it helps:)

Step-by-step explanation:

23% of 25,314 = 25,314 x .23 = 5822 then add that to the original population. 25,314 + 5,822 = 31,136

(wateroflife18, brainly.com)

What is the solution to the system of equations shown?
y = 2x + 2
y = 4x - 2

A. (2, 6)
B. (2, -6)
C. (-2,-6)
D. (-2, 6)

Answers

Answer:

A. (2, 6)

Step-by-step explanation:

Hope it helps you in your learning process

What is being distributed in the following problem?
5 - 31d + 7)

Answers

Answer:

the -3 is being distributed. --> the 5 stays outside till the end. multiply the 3 inside first, then the negative. and then add the 5 to the final answer            

             5-3(d+21)  = -->> 5-(3d+21)=--->>>5-3d-21=3d-16

Step-by-step explanation:

5-3(d+21) =

5-(3d+21)=

5-3d-21=

ans=3d-16

I WILL GIVE BRAINLIEST; PLEASE ANSWER ASAP

Answers

Answer:

5

Step-by-step explanation: since i helped can i have brainlist please :D

What is the probability that a student selected at random takes math or drama? (round answer to 3 decimal places)

Answers

Distribution table for the question is attached below :

Answer:

0.533

Step-by-step explanation:

Let :

M = math

D = Drama

Probability that a randomly chosen student takes math or drama ; P(M u D)

Recall:

P(A u B) = p(A) + p(B) - p(A n B)

P(M u D) =  p(M) + p(D) - p(M n D)

Probability = required outcome / Total possible outcomes

From the table :

P(M) = 119 / 302

P(D) = 67 / 302

P(M n D) = 25 / 302

Therefore,

P(M u D) = 119/302 + 67/302 - 25/302

P(M u D) = (119 + 67 - 25) / 302

P(M u D) = 161 / 302

P(M u D) = 0.5331125

P(M u D) = 0.533 ( 3 decimal places)

At a certain toy store, tiny stuffed
pandas cost $3.50 and giant stuffed
pandas cost $14. If the store sold 29
panda toys and made $217 in revenue
in one week, how many tiny stuffed
pandas and giant stuffed pandas were
sold?

Answers

Answer:

130

Step-by-step explanation: add

a man goes 10 meter due east and 24 meter dues north. what is the distance from the starting point​

Answers

Answer:

26m

Step-by-step explanation:

I think he is 34 meters away from the starting distance because 10+24=34

i need help attachment below will give brainliest

Answers

Answer:

see explanation

Step-by-step explanation:

(1)

Calculate the circumference (C) of the wheel

C = πd (d is the diameter )

   = 3.14 × 8 = 25.12 cm

Now divide the distance travelled by the circumference.

times rotated = 12560 ÷ 25.12 ≈ 500

----------------------------------------------------------------

(2)

The area (A) of a circle is calculated as

A = πr² ( r is the radius )

Find r by using the circumference (C)

2πr = 50.24 ( divide both sides by 2π )

r = 50.24 ÷ 6.28 = 8 , then

A = 3.14 × 8² = 3.14 × 64 ≈ 200.96 ft² ( student 3 )

-----------------------------------------------------------------

(3)

C = πd = 3.14 × 30 = 94.2 ≈ 94 m

+
3
0
1
4
5
15
2
-1
10
-3
14 13
19
6
12 11
+8
-7
-5
-4
0) Which expression is equivalent to-4-9?
4 + 9
4
9
–4+ (-9)
-4 +9
In

Answers

Answer:

-4 + (-9)

Step-by-step explanation:

It doesn't matter if you add the plus sign, -4 + (-9) is the same as -4-9

Other Questions
find the exact value of tan30 cos30give your answer in its simplest form Looking at the table, how do you think Warren collected this data? Take 25 points but help me Please read the text and answers the questions in the picturethe text(Three children who had been missing for two days have been found safe and well. The three ten - year olds, two boys and a girl, disappeared after school on Wednesday. Police had issued photographs of the three, and had been searching nearby houses. They were eventually spotted by a neighbor, who alerted the police. They said they had slept out in a garden shed for a dare, and hadn't realized the concern they had caused) Make up a short story about a character who's so afraid of making thewrong choice that their indecision ruins the very thing they were hoping tokeep safe. Please, this is due soon.After finding out the truth about fast fashion... What did you use to think about it, what did the media say about it (what is fast fashion?), after finding out the truth what do you think now and how will that change your worldview? How is relative location often described? pls look at the picture and answer Solve the system:y = -2x + 9y = 3x + 9 8. Find the length of the unknown side. Round youranswer to the nearest tenth.10 ftA. 13.5 AB. IfC. 4.4 ftD. 19 ft Marcus Aurelius had the greatest influence on the Roman Empire because he cared about all Roman citizens. Im confused.Help plz S Where did pre-Colombian Native Americans live? In which sentence does a prepositional phrase modify an adverb?a.The temperature really dropped after sundown.b.Did you drink hot cocoa for the warmth?c.Late in the night I got an extra blanket.d.My parents built a fire in the wood stove. Seth's bill for breakfast at a restaurant was $ 79. The chef took a 20% tip. What was the amount of the tip?pls help WHICH EQUATION BEST REPRESENTS? What are some actions that an unlimited government might take to maintain its authority to lead a nation? The book is " TO KILL A MOCKINGBIRD." someone please help me-- hey! please help me with this question 7+89-24 mins after a couple hours I got the wrong order for a few days after A speaker says: You know, we're gonna have to talk about this later. Which of the following is the correct way to transcribe this according to our Clean Verbatim Style Guide? A) We're gonna have to talk about this later. B) We're going to have to talk about this later. C) You know, we're going to have to talk about this later.